Đến nội dung

fghost nội dung

Có 223 mục bởi fghost (Tìm giới hạn từ 29-04-2020)



Sắp theo                Sắp xếp  

#292306 Ôn thi Olympic Toán học sinh viên 2015 [Giải tích]

Đã gửi bởi fghost on 05-01-2012 - 16:24 trong Thảo luận về các kì thi, các kì kiểm tra Toán sinh viên

Bài 2:
Vài nhận xét:
1/ Hàm liên tục phải thõa mãn intermediate value property (tính chất "trung điểm", cụ thể nếu $f(a) < f(b)$ và $a < b$ thì f nhận mọi giá trị trong $(f(a), f(b))$, tức là với mọi c mà $f(a) < c < f(b)$ tồn tại $y\epsilon (a,b)$ sao cho $f(y)=c$).
2/ Hàm số đơn ánh 1-1 không thể đạt 2 giá trị bằng nhau.

Với nhận xét 2/ giả sử f không tăng hay giảm nghiêm ngặt, thì sẽ tồn tại 3 điểm $a <x<y<z<b$ sao cho $f(x)<f(y)>f(z)$ hay $f(x)>f(y)<f(z)$. Nếu $f(x)<f(y)>f(z)$ xảy ra, ta có vì f thõa mãn intermedidate value property nên f phải đạt mọi giá trị trong $(f(x), f(y))$ bằng $(x,y)$ và mọi giá trị trong $(f(z), f(y))$ bằng $(y,z)$, và hiển nhiên tồn tại một giá trị c nào đó mà $f(x)<c<f(y)$ và $f(z)<c<f(y)$ sẽ đượt đạt bởi 2 giá trị từ 2 đọan khác nhau $(x,y)$ và $(y,z)$ nên f sẽ không phải đơn ánh. Tương tự với trường hợp còn lại.

Bài giải "chấp nhận" hơi nhiều thứ quá.



#453802 Chứng minh định lý Lớn Fermat với kiến thức PT

Đã gửi bởi fghost on 29-09-2013 - 08:08 trong Toán học hiện đại

Cám ơn bạn đã đọc bài cm của mình.

Giả sử u + v không chia hết cho n, theo bổ đề 5a thì biểu thức A = ... không chia hết cho n và từ (4) thì $t^{n}$ không chia hết cho n, từ đó t không chia hết cho n, mâu thuẩn với giả sử t chia hết cho n. Đó là điều bạn đang quan tâm đấy!

 

Ah, ra vậy. Mình không để ý đến BĐ 5a.




#453781 Chứng minh định lý Lớn Fermat với kiến thức PT

Đã gửi bởi fghost on 29-09-2013 - 04:13 trong Toán học hiện đại

Đầu trang 10, trường hợp 2.1.2. Mình đọc thì hiểu như sau

 

. Trong 2.1, ta giả sử, $n$ nguyên tố lớn hơn 3. Và ta có đẳng thức (4) như sau

$$(u+v)\sum_{i=0}^{n-1}(-1)^iu^{n-i-1}v^i= t^n$$

. Sau đó đến trường hợp 2.1.2, ta giả sử thêm, trong $u,v,t$ chỉ có $t \vdots n$. Trong file pdf có ghi "ta giả sử $t \vdots n$, từ (4) suy ra $(u+v) \vdots n$".  

 

Câu hỏi của mình: Vì sao chúng ta suy ra được $(u+v) \vdots n$?

 

Phân tích nguyên tố của $t^n$ sẽ có dạng $e_1^ne_2^n\dots e_k^n n^n$ với $e_i$ và $n$ nguyên tố. Hình như không có lý do $n^n$ không thể hoàn toàn nằm trong $\sum_{i=0}^{n-1}(-1)^iu^{n-i-1}v^i$. 

 

Thật sự mình cũng chỉ nhìn sơ qua vậy thôi, có lẽ mình đã bỏ xót gì đấy.




#303168 $\text{Cho}\,\left | G \right |= 2n; H\leqslant...

Đã gửi bởi fghost on 09-03-2012 - 20:56 trong Đại số đại cương

Rất chính xác.

Em suy luận đến bước "Nếu $x \in H \Rightarrow x^2 \in H$" là đúng, vì H là nhóm, nên bảo tồn phép toán của nhóm (x là phần tử của H nên x*x phải thuộc H).

Việc em nhận xét được $H \triangleright G$ là rất chính xác và quan trọng.

Nhưng vấn đề là khi $x \notin H$ ta biết x phải thuộc lớp còn lại (hay đối tập còn lại), nhưng rõ ràng là đối tập còn lại không phải là nhóm. Bước này của em không đúng "Nếu $ x \notin H \Rightarrow x\in xH\Rightarrow x^2 \in xH=H $" ($x \in xH$ nhưng không thể suy ra $x^2 \in xH$ vì $xH$ không phải là nhóm; và $x\notin H$ thì $xH \ne H$).



#303491 $\text{Cho}\,\left | G \right |= 2n; H\leqslant...

Đã gửi bởi fghost on 11-03-2012 - 08:29 trong Đại số đại cương

Bài 1)

Chứng minh A là nhóm con của G:

Trước tiên: $A \neq \varnothing $ vì $ \exists e \in G: e^m=e \Rightarrow e \in A $

Lấy $ {x_1}^m, {x_2}^m $ thuộc A, xét: ${x_1}^m.[{x_2}^m]^{-1}=(x_1.{x_2}^{-1})^m={x_0}^m \in A $ (vì $ x_0 = x_1.{x_2}^{-1} \in A $ )


$x_0^m \in A$ vì $x_0=x_1.x_2^{-1} \in G$ theo định nghĩa của A mà $x_0^m \in A$

Vậy $ A \leqslant G $.

Tương tự ta cũng cm được B là nhóm con của G.

Ta có: $ A \cap B= \left \{ x \in G: x^m=e \wedge x^n=e \right \}=\left \{ e \right \} $

Thật vậy: nếu $ x \neq e $ mà $ x^m=e\wedge x^n=e \Rightarrow x^m = x^n \Rightarrow x^{m-n}=e \Rightarrow (m-n) \vdots m \wedge (m-n) \vdots n \Rightarrow $ vô lí, vì (m,n)=1.


Tại sao $x^{m-n}=e \Rightarrow (m-n) \vdots m \wedge (m-n)\vdots n$? $m,n$ không phải là cấp của $x$.

Theo anh nghĩ, lý luận đúng hơn phải như thế này.

Vì $G$ là nhóm hữu hạn, với $x\ne e$, $1<|x| < \infty$ và $x \in A\cap B$ thì $x^m=e$ và $x^n=e$ dẫn đến, $m \vdots |x|$ và $n \vdots |x|$. Như vậy, $(m,n) \geq |x| > 1$ vô lý.

Ta có: $ AB = \left \{ ab: a\in A \wedge b\in B\right \} $
Hiển nhiên $ AB \neq \varnothing $

Ta cm $ AB \subset G $: Lấy x thuộc AB, suy ra tồn tại a, b lần lượt thuộc A, B sao cho $ x = ab $. Vì a, b thuộc A, B nên suy ra a và b cũng thuộc G, do đó $ x = ab \in G $

Ta cm $ G \subset AB $: (...chưa cm được!)


Chiều $AB \subset G$ rất đúng.

Để chứng minh $AB=G$ em có thể dùng $|AB|= \frac{|A|.|B|}{|A\cap B|}=mn=|G|$. Hay nếu em muốn cho thấy $G \subset AB$ thì em phải chứng minh được mọi phần tử của $G$ phải là tích của phần tử nào đó của $A$ và $B$. Điều này đơn giản nếu em biết định lý cơ bản của nhóm Abel hữu hạn, chỉ cần áp dụng, sẽ ra kết quả. Coi như không biết, thì phải giải làm sao?

$x \in G \Rightarrow mn=|G| \vdots |x|$ Nhưng vì $(m,n)=1$, $m \vdots |x|$ hay $n \vdots |x|$. Giả sử $m\vdots |x|$ thì $x^m=e$ nên $x\in A$. Trường hợp còn lại $n \vdots |x|$ thì $x^n=e$ nên $x \in B$. Mà rõ ràng $A \subset AB$ và $B \subset AB$ nên trong trường hợp nào thì $x \in AB$. Như vậy $G \subset AB$.



#305342 $\text{Cho}\,\left | G \right |= 2n; H\leqslant...

Đã gửi bởi fghost on 19-03-2012 - 20:00 trong Đại số đại cương

1 ở đây là $e$ đấy em. Không có gì khác biệt.

$S$ có tối đa $2n$ phần tử (do là tập con), nhưng vì $S$ không có $e$ nên $S$ có tối đa $2n-1$. Nhưng vì $S$ phải có chẵn số phần tử, nên $S$ ko thể có $2n -1$ mà phải là $2n -2$.

Vì $S$ có tối đa $2n -2$ phần tử, nên $G$ có tối thiểu 2 phần tử không thuộc $S$ tức là có tối thiểu 2 phần tử mà nó là nghịch đảo chính nó. Loại trừ $e$ ra, thì còn lại 1 phần tử là nghịch đảo của chính nó, mà phần tử $e$ là duy nhất trong 1 nhóm, nên $x \ne e$, nên phần tử x này có cấp 2.



#305221 $\text{Cho}\,\left | G \right |= 2n; H\leqslant...

Đã gửi bởi fghost on 19-03-2012 - 07:34 trong Đại số đại cương

Thật ra ý tưởng bài 2 rất đơn giản. Một phần tử cấp 2, không phải đơn vị, sẽ là nghịch đảo của chính nó. Ví dụ $x^2=1 \Leftrightarrow x=x^{-1}$. Như vậy, khi ta xem xét tập $S$ là tập của những phần tử khác với nghịch đảo của chính nó $y\ne y^{-1}$. Rõ ràng, với mọi phần tử của $S$ và phần tử nghịch đảo của nó sẽ đi theo cặp. Như vậy số phần tử của $S$ phải là số chẵn. Em phải tự thuyết phục mình số lượng phần tử của $S$ là số chẵn.

Mà, phần tử đơn vị, 1, rõ ràng không nằm trong $S$ như vậy thì số lượng phần tử của $S$ (ta đã biết là số chẵn) tối đa phải là $2n-2$. Và như vậy tồn tại phần tử cấp 2.



#304480 $\text{Cho}\,\left | G \right |= 2n; H\leqslant...

Đã gửi bởi fghost on 15-03-2012 - 21:50 trong Đại số đại cương

Anh nói hơi lộn xộn. Anh thấy phần $AB=G$ có 2 cách đi. Cách 1: chứng minh $AB \subset G$ và $G \subset AB$. Cách 2: như em nói rất chính xác, $AB \subset G$ hay $G \subset AB$, và $|AB| = |G|$.

Bài 2:
Đầu tiên, nhận xét: $|x| =2 \Leftrightarrow x^2=1 \text{ và } x \ne 1 \Leftrightarrow x=x^{-1} \text{ và } x\ne 1$.

Sau đó, em tính số phần tử của tập $S= \{ x \in G: x \ne x^{-1} \}$. Chú ý số phần tử của $S$ là hữu hạn nguyên dương, nên chỉ có thể là chẳn hay lẻ.

Cuối cùng suy ra sự tồn tại của phần tử có cấp 2.

Cách giải này anh tham khảo từ trên mạng. Nếu em đã học đến định lý Cauchy cho thấy nếu $p$ nguyên tố là ước của cấp của nhóm $G$ thì tồn tại phần tử có cấp $p$ hay những định lý Xi-lo (Sylow) thì bài này không cần chứng minh.



#303059 $\text{Cho}\,\left | G \right |= 2n; H\leqslant...

Đã gửi bởi fghost on 09-03-2012 - 11:06 trong Đại số đại cương

Các pro ơi, giúp e giải bài này nhé, mặc dù đã đọc rất nhiều lần về lý thuyết nhóm, nhưng vẫn ko tự giải được bài nào cho ra hồn cả, hic..

Cho nhóm G hữu hạn có 2n phần tử, H là nhóm con của G, H có n phần tử. CMR: $\forall x \in G\Rightarrow x^2 \in H.$

$\left | G \right |= 2n; H\leqslant G; \left | H \right |= n. CMR: \forall x \in G\Rightarrow x^2 \in H.$


Bài này có thể giải thế này, còn cách nào ngắn hơn không thì mình không biết.

Cần biết điều sau:
1/ Nếu H, K là nhóm con của G, và nếu H normal (nhóm con chuẩn tắc), thì HK là nhóm con của G, với $HK=\{hk: h \epsilon H \text{ và } k \epsilon K\}$. Hơn thế $|HK| = \frac{|H||K|}{|H\cap K|}$.
2/ Định lý cơ bản của nhóm xích: 1 nhóm xích của bậc d, thì nó có đúng 1 nhóm xích con có bậc ứng với mỗi thừa số p của d, và mỗi nhóm xích con có bậc p đó được sinh bởi $x^{d/p}$.

Nhận thấy, $|H| = 1/2 |G|$, H phải normal (chuẩn tắc) vì chỉ có thể có 2 đối tập (coset) trong G, là H và aH với a không thuộc H.

Với bất kì x trong G. Gọi $K=<x>$. Rõ ràng K là nhóm con của G, K Abel (Abelian) và là nhóm xích (cyclic).

Xem xét $HK$ là nhóm con của G mà $H \subset HK$ nên HK phải là H hay chính là G. Mà $|HK|=\frac{|H||K|}{|H\cap K|}=\frac{n|K|}{|H\cap K|}$. Nếu $|HK|=|H|=n$ thì $|K|=|H\cap K| \Rightarrow K \subset H \Rightarrow x^2 \epsilon H$.

Nếu $|HK|=|G|=2n \Rightarrow |K|=2|H\cap K|$ mà $H\cap K$ là nhóm con của K. Bây giờ ta có K là nhóm xích $\{x, x^2, x^3,...., x^{|K|}\}$, với $|K|$ chẵn, mà nhóm con duy nhất của K có bậc $\frac{|K|}{2}$ chính là nhóm xích được sinh bởi $x^2$.

Như vậy $<x^2> = H\cap K$ cho nên $x^2 \epsilon H$.



#303926 $\text{Cho}\,\left | G \right |= 2n; H\leqslant...

Đã gửi bởi fghost on 13-03-2012 - 09:17 trong Đại số đại cương

Cách anh cm $ A \cap B = \left \{e\right\} $ e đã hiểu.

Anh cho e hỏi: Nếu $ \left | AB \right |=\left | G \right | $ thì ta có thể suy ra $ AB=G $ hả anh?

Em ko chắc cách hiểu của e là đúng nên hỏi anh: $ A \subset AB $ là do nếu $ x \in A \Rightarrow x=xe $ nên $x \in AB $ ?


Vì $AB \subset G$ em đã chứng minh, $|G| < \infty$ nên khi số lượng phần tử của $AB$ bằng với của $G$ thì 2 bên phải bằng nhau (em có thể tự thuyết phục mình bằng phản chứng trên số phần tử của tập phần bù của $AB$ trong $G$, với $|G \setminus AB|=0$)

"$ A \subset AB $ là do nếu $ x \in A \Rightarrow x=xe $ nên $x \in AB $" chính xác.



#473358 Tính $ ( e - x )^{-1} $

Đã gửi bởi fghost on 28-12-2013 - 09:05 trong Đại số đại cương

Anh ơi, thế còn chứng minh $ ( e - x ) $ khả nghịch thì có phải là do $A$ là vành và có đơn vị là $e$ không ạ?

 

Khi đó $ ( e-x ) . y =e \Rightarrow y =\dfrac{e}{e-x} $ với $ y$ là phần tử nghich đảo của $ (e -x) \Rightarrow (e-x)^{-1} = y = \dfrac{e}{e-x}$ có phải không ạ ?

 

Chứng minh $e-x$ khả nghịch, tức là chứng minh tồn tại phần tử $y$ nào đó sao cho $xy=e$, nói cách khác, phần tử $(e-x)^{-1}$ tồn tại (trong vành $A$ - vành $A$ giao hoán). Trong phép tính trên của mình, mình đã chỉ ra phần tử nghịch đảo của $(e-x)$ chính là

$$e+x+x^2+\dots+x^6$$

Vì $A$ là vành nên phần tử trên tồn tại trong vành $A$

Có phải A là vành thì đợn vị  $ e =1 $  không ạ ?
Anh ơi, cho em hỏi : $ 0 $ ở đây có phải là phần tử trung hòa không ạ ?
và $ x^7 $ ở đây, có phải là  $ x^7 = x.x.x.x.x.x.x $  không ạ?
và phép . ở đây có phải là phép nhân thông thường trong $ R $ không ạ ?

$e$ hay $1$ chỉ là kí hiệu thôi. $0$ là phần tử trung hòa cho phép tính $+$, và 1 là phần tử trung hòa cho phép tính $.$ $x^7=x.x.\dots.x$. Phép nhân là phép nhân được xác định trong vành $A$.




#473595 Tính $ ( e - x )^{-1} $

Đã gửi bởi fghost on 29-12-2013 - 07:35 trong Đại số đại cương

Bạn hiểu chính xác, phần tử $x$ khi được nhân $7$ lần, sẽ ra phần từ trung hòa $0$ (của phép tính cộng).

$$x.x.x.x.x.x.x=0$$

Tất nhiên, điều này không bắt buộc là $x=0$.




#473341 Tính $ ( e - x )^{-1} $

Đã gửi bởi fghost on 28-12-2013 - 00:59 trong Đại số đại cương

Thay $e=1$ cho dễ nhìn.

 

$$1=1-x^7=(1-x)(1+x+x^2+\dots+x^6) \Rightarrow (1-x)^{-1}=1+x+x^2+\dots+x^6$$




#473420 Tính $ ( e - x )^{-1} $

Đã gửi bởi fghost on 28-12-2013 - 14:03 trong Đại số đại cương



Thế $ 1 -x^7 = ( 1-x )( 1+x+x^2+.. + x^6) $, đều là kí hiệu hết ạ ?

 

Nếu không thay $ e =1 $ thì có phải là thế này không ạ?

 

$ e = e- x^7 =  \left( \sqrt[7] {e} \right)^7 - x^7 $

 

Nếu kí hiệu phần tử trung hòa của phép nhân $.$ là $e$ thay vì là $1$, thì ta cũng có

$$(e-x)(e+x+x^2+\dots+x^6)=e-x^7$$

Thực hiện phép tính tương tự như đại số thông thường. Nếu nghĩ kĩ, thì khi khai triển vế trái, ta chỉ dùng những tính chất có sẵn trong vành $A$ (như $e.x=x$, tính phân phối giữa phép cộng và phép nhân,..).




#474699 Tính $ ( e - x )^{-1} $

Đã gửi bởi fghost on 02-01-2014 - 11:49 trong Đại số đại cương

Ta chứng minh $x.0=0$ trước. Dễ thấy

$$x.0=x.(0+0)=x.0+x.0 \Rightarrow 0=x.0$$

 

Sau đó, ta có

$$x.x+x.(-x)=x.(x+(-x))=x.0=0 \Rightarrow -(x.x)=x.(-x)$$




#473888 Tính $ ( e - x )^{-1} $

Đã gửi bởi fghost on 30-12-2013 - 03:02 trong Đại số đại cương

Theo định nghĩa, vành $A$ (giao hoán) là nhóm $A$ (với phép cộng giao hoán, phần tử trung hòa $0$), và kết hợp thêm với phép nhân (phần tử trung hòa $1$) có tính chất kết hợp (giao hoán) và, phù hợp với tính chất phân phối với phép cộng.)

 

Phép cộng (hay trừ) trong vành $A$ mà mình nói trong những bài trên là phép tính có sẵn trong định nghĩa của $A$, và có phần tử trung hòa là $0$.

 

Những phép toán trong $A$ chưa chắc là phép tính trong $R$ nhưng khi thực hiện những khai triển trên, thì ta chỉ dùng tính chỉ dùng tính chất của $A$ mà thôi.

 

Nếu bạn có thể liệt kê lại định nghĩa của vành $A$, mình có thể viết ra từng dòng phép tính để cho thấy phép tính của chúng ta thực hiện được trong vành $A$.




#474356 Tính $ ( e - x )^{-1} $

Đã gửi bởi fghost on 01-01-2014 - 03:44 trong Đại số đại cương

Đầu tiên ta nên hiểu $e-x^7=e+(-x^7)=e+(-0)$ với $-0$ là phần tử "đối" của $0$ với phép cộng trong vành $A$. Ta thấy $0+0=0$, cho nên phần tử đối của $0$ trong phép cộng phải là $0$, cho nên $-0=0$. Do đó, $e-x^7=e+(-0)=e+0=e$. $e$ là phần tử trung hòa của phép nhân, bất kì phần tử nào khi cộng với $0$ đều bằng chính nó, kể cả $e$.




#473904 Tính $ ( e - x )^{-1} $

Đã gửi bởi fghost on 30-12-2013 - 11:14 trong Đại số đại cương

Chứng minh của bạn đúng rồi.




#465359 Nghịch lý monty hall

Đã gửi bởi fghost on 19-11-2013 - 21:29 trong Nghịch lý



mình nghĩ ở đây mình không đề cập đến việc có đổi quà hay là không, bản chất nó là xác định xác suất CÓ QUÀ của từng hộp A và B sau khi MC mở hộp rỗng C kia thôi.

 

Mình thấy bài đầu của bạn có câu này

 

 

Hỏi người chơi có nên đổi hộp không.

 

Việc cho phép người chơi đổi quà làm thay đổi xác suất có quà (như giải thích bên dưới). Còn nếu người chơi không được phép đổi, hay bắt buộc phải đổi, thì xác suất sẽ khác đi.

 



mình có đính chính lại như sau:

-giả sử nếu ko đổi hộp thì xác suất trúng cũng chỉ là $\frac{1}{3}$ bời vì đơn giản mc chỉ bỏ đi 1 hộp ko có quà ko phải hộp mình chon nên hôp chọn được bảo toàn$\Rightarrow$ xác suất được bảo toàn

-còn về xác suất khi đổi thì vẫn khẳng định xác suất là $\frac{2}{3}$.Đây có thể là cách giải đễ hiểu hơn cách giải trước:

+tương tự ta có 3 hộp: k,k,c.

+theo đề bài thi khi mc bỏ đi 1 hộp k thì nếu trước đó ta chọn hộp k còn lại thì khi đổi ta chắc chắn được hộp c.vậy để tính xác suất khi đổi ta được hộp c thì phải tính  xác suất ta chọn được hộp k lúc đầu.

+xác suất chọn lúc hôp k khi mc chưa lấy hộp là $\frac{2}{3}$

+nên nếu mc lấy đi 1 hộp k rồi thì khi đổi ta chắc chắn được hộp c.

Vậy xác suất khi đổi hộp là $\frac{2}{3}$

 

Bạn giải thiếu rồi hay có lẽ đề không rõ và mình hiểu nhầm, người chơi có quyền đổi hay không đổi hộp (?) sau khi MC mở hộp rỗng.

 

Bạn đúng ở chỗ nếu chọn đổi, thì xác suất lúc đầu để chọn hộp k là $2/3$. Nhưng xác suất đổi là $1/2$. Nên xác suất nếu chọn đổi và được quà là $1/2*2/3$. Còn nếu bạn không đổi ($1/2$), và bạn được quà, nên lúc đầu phải chọn hộc c ($1/3$), vì vậy xác suất là $1/2*1/3.$ Tổng xác suất để được quà là $1/2.$

 

Tất nhiên, nếu người chơi bắt buộc phải đổi quà, thì bạn đã đúng và đủ.

 

Đọc lại đề thì có câu này

Nhưng sau khi đổi thì xác suất của cái hộp bị mở là bằng 0

 

Như vậy tức là người đã đổi,




#465442 Nghịch lý monty hall

Đã gửi bởi fghost on 20-11-2013 - 10:52 trong Nghịch lý



về phần mình thì mình sẽ chọn đổi để có nhiều cơ hội để trúng hộp có hơn.

Đổi hay không đổi thì cơ hội trúng cũng bằng $1/2$ mà thôi.

 

Chỉ cần bạn nghĩ lại, lúc đó chỉ còn 2 hộp. Bạn không biết hộp nào có quà, việc đổi hay không đổi hoàn toàn giống với việc chọn mới mà thôi.

 

(Nhưng vì sao trong bài của mình, mình lại nói bạn đã giải đúng và đủ, và cho xác suất là $2/3$ sau khi đổi? Là bời vì đó là xác suất trúng với điều kiện người chơi đã đổi. Hiển nhiên không thể so sác xuất có điều kiện đó với xác suất trúng của quyết định đổi hay không đổi được).




#464754 Nghịch lý monty hall

Đã gửi bởi fghost on 17-11-2013 - 06:14 trong Nghịch lý

Xác suất có quà của A và B là như nhau và bằng $1/2$ vì mỗi hộp đều có khả năng có quà như nhau.

 

Nghĩ theo chiều hướng như thế này sẽ dễ hiểu hơn: bạn chọn 1 hộp trong 3 hộp rỗng. Mình sẽ để quà vào 1 trong 3 hộp đó ngẩu nhiên, thì xác suất hộp bạn chọn có quà là $1/3$. Vì MC sẽ loại bỏ 1 hộp không có quà, nên mình chỉ có 2 lựa chọn để đặt quà vào, và vì mình chọn ngẫu nhiên, nên xác suất có quà của bạn sẽ là $1/2$.




#465206 Nghịch lý monty hall

Đã gửi bởi fghost on 19-11-2013 - 08:30 trong Nghịch lý



Tôi lại có cách nhìn hơi khác. Giả sử ba hộp là A,B,C. Như vậy ban đầu xác suất có quà của mỗi hộp đều bằng 1/3.

Ban đầu ta chọn một hộp, giả sử là A, thế thì trong B và C luôn có ít nhất 1 hộp không có quà và người dẫn chương trình (MC) biết chắc điều này! MC mở hộp không có quà, giả sử đó là C. Bây giờ MC hỏi ta có đổi A với B hay không?

Thực ra nếu ta có đổi thì cũng chỉ là đổi cái xác suất 1/3 này lấy cái xác suất 1/3 kia mà thôi!

Nếu được quyền đổi, thì sau khi loại bỏ 1 hộp, thì chỉ còn 2 khả năng mà hộp quà ta chọn có quà (đổi hay không đổi: Nếu hộp A có quà, mà ta đổi, thì không được quà. Nếu hộp A có quà, mà ta không đổi thì ta được quà). Và vì việc đổi hay không đổi có xác suất xảy ra như nhau, nên xác suất hộp ta chọn có quà (sau khi quyết định đổi hay không đổi) là $1/2$. 

 

Còn nếu như không được quyền đổi, thì như bạn hxthanh đã đề cập, xác suất được quà là $1/3$ vì việc mở hộp không quà chỉ đơn giản là hành động kiểm tra mà thôi. (mình đã suy luận sai trong lần đầu)




#530579 Chứng minh dãy khớp ngắn

Đã gửi bởi fghost on 26-10-2014 - 10:28 trong Đại số tuyến tính, Hình học giải tích

sr,em đọc nhầm thành đẳng cấu, vậy đồng cấu là gì ạ.

 

nói ra ngại, mình không biết đồng cấu là gì cả vì mình không học theo từng vựng tiếng việt :D mình thấy bài dùng từ đó nên mình dùng theo.

 

(đồng cấu theo mình đoán có lẽ là homomorphism - trong trường hợp này là homomorphism giữa 2 không gian vector, nói cách khác đó là ánh xạ tuyến tính. Bạn có thể đã thấy homomorphism - đồng cấu - trong lý thuyết đại số trừu tượng, ánh xạ giữa 2 nhóm giao hoán, hay 2 vành giao hoán thõa mãn 1 vài tính chất đặc biệt. Về 1 mặt nào đó, không gian vector chỉ là 1 nhóm giao hoán có thêm cấu trúc nhân với 1 trường).

 

nói nhiều thành ra nói dở, đối với bài này, ta chỉ cần chứng minh tồn tại 1 ánh xạ tuyến tính $\bar{f}$ như trên.




#530492 Chứng minh dãy khớp ngắn

Đã gửi bởi fghost on 25-10-2014 - 20:52 trong Đại số tuyến tính, Hình học giải tích

$$0\rightarrow \ker f_2\rightarrow V_2\rightarrow \text{Im}f_2\rightarrow 0$$

 

Ở dòng này thì toán tử tuyến tính từ $\ker f_2$ vào $V_2$ là gì ạ

 

$\ker f_2 \subset V_2$, toán tử bạn cần chỉ là $x \mapsto x$

 

Còn vì sao chỉ cần chứng minh trên dãy ngắn. Có lẽ quy nạp sẽ dễ thấy. Nếu bạn có dãy khớp $0 \rightarrow V_1 \rightarrow \dots \rightarrow V_{n-2} \overset{f_{n-2}}{\rightarrow} V_{n-1} \overset{f_{n-1}}{\rightarrow} V_n \rightarrow 0$, bạn có thể tách nó ra thành 2 dãy

$$0 \rightarrow V_1 \rightarrow \dots \rightarrow V_{n-2} \overset{f_{n-2}}{\rightarrow} im f_{n-2} \rightarrow 0$$

$$0 \rightarrow \ker f_{n-1} \rightarrow V_{n-1} \rightarrow V_n \rightarrow 0$$

 

Dùng giả thuyết quy nạp sẽ được kết quả mong muốn.

 

Vì mọi dãy khớp đều có thể bị bẽ gảy thành dãy khớp ngắn, tương tự như trên.

 

Sorry bạn leminhansp vì mình nhảy vào giữa chừng :)




#530546 Chứng minh dãy khớp ngắn

Đã gửi bởi fghost on 26-10-2014 - 01:19 trong Đại số tuyến tính, Hình học giải tích

tại sao bạn muốn chứng minh đó là đơn ánh? 

 

$\bar{f}$ của bạn có $\ker$ đâu phải bằng $0$.

 

để chứng minh tồn tại 1 đồng cấu $\bar{f}$ như vậy, bạn chỉ cần chứng minh với mọi 2 phần tử $v, v'$ sao cho $v - v' \subset V'$, thì ảnh của chúng dưới $f$ (hay $\bar{f}$) giống nhau. tức là ánh xạ của bạn có nghĩa. và sau đó, nó là đồng cấu thì hiển nhiên hơn.